Jump to content

Devridaim ve termodinamik hakkında ne biliyorsunuz?


Recommended Posts

1 saat önce, deadanddark yazdı:

Kuantum düsünce teknigi(!!!) ile mi calistiracaksin sistemi?

 

1 saat önce, Sütlü Kase yazdı:

Umuyorum ki youtube dan yarı bozma bir şey gelmez buraya

 

Bir saat önce, deadanddark yazdı:

Stres oyuncaklarindan birini de getirebilir. Newtonun Besigi gibi.

 

Hayır değil. Sizlere çok daha iyi matematiksel veriler sunmak ve sunumunu kolaylaştırmak için çok ayrıntılı programatik ve görsel simülasyonunu oluşturuyorum. Zaten bu sistemin sanayide 10 m3'lük bir prototipini hazırladık. Şimdi ölçümleri daha kaliteli yapabilmek için 3D yazıcı ile daha küçük masaüstü bir prototipini yapmayı planlıyoruz. Yapacağım programatik ve görsel simülasyon bunu daha iyi yapmamızı sağlayacak. Bu sistemin çalışması ile ilgili aramızdaki yüksek makine mühendisleri dahil en ufak bir tereddüdümüz yoktur. Sadece daha verimli hale getirmek için neler yapabiliriz. Bu amaçla sizlerle paylaşmayı planlıyorum.

Link to post
Sitelerde Paylaş
  • İleti 318
  • Created
  • Son yanıt

Top Posters In This Topic

Arkadaşlar @Sütlü Kase isimli arkadaşa yazdıklarımdan sonra benimle jartiyer pazarlığına girecek aranızda matematik ve fizik konusunda kendisine güvenen arkadaşlar yok mu? Ben bahsettiğim konuyla çok ilgiliyim ve sonuna kadar sizin dimağınıza durgunluk verecek gerçekten mükemmel bir projeden bahsediyorum. Siz tüm gerçekleri öğrendikten sonra onunla yüzleşecek cesareti gösteremeyip benimle olmadık söz dalaşına gireceksiniz diye endişe ediyorum. Halbu ki her şey fiziksel ve matematiksel olarak ortada olacak. Yazdıklarınıza bakılırsa bunun olabilirliğine hiç ihtimal vermiyorsunuz ve bana inceden o küçük beyinlerinizle ayar vermeye çalışıyorsunuz. Kendine güvenen varsa çıksın ortaya, bekliyor olacağım.

 

O halde tümünüze soruyorum. 

 

Ben bunu fiziksel ve matematiksel olarak kanıtladığımda tüm vücüdundaki kılları ağdalayıp, sonra nemlendirici ile yumuşatıp, dantelli bir kilot giyip üzerinde pembe ve ipek bir straplez giyerek karşıma çıkacak mısın? Bunu yapamadığımda aynını kabul ediyorum. Buraya fotoğrafını koyarım. Fakat sizlerden kendisine bu temel bilgilerde güvenen birilerini arıyorum. Öyle hesap yapmaktan kaçınıp işkembeden sallayacaksanız hiç uğraşmayayım.

tarihinde John_Ahmet tarafından düzenlendi
Link to post
Sitelerde Paylaş
On 08.02.2019 at 02:46, John_Ahmet yazdı:

@Sütlü Kase Ben bunu fiziksel ve matematiksel olarak kanıtladığımda tüm vücüdundaki kılları ağdalayıp, sonra nemlendirici ile yumuşatıp, dantelli bir kilot giyip üzerinde pembe ve ipek bir straplez giyerek karşıma çıkacak mısın? Kısaca dürüst müsün? Savlarının arkasında duruyor musun? Sen bu gerçeklerle yüzleşebilecek kadar fiziğe, matematiğe ve bilime güveniyor musun? Dahası bu konularda kendini yeterli görüyor musun?

 

Ben bu konudaki bir gelişmenin insanlık için çok büyük bir ihtiyaç olduğunu düşünüyorum. Böyle bir gelişme tüm savaşları bitirebilir ve daha pek çok konuda faydalı işler yapmamızı sağlayabilir. Bu nedenlerle konuyu anlayabilecek kadar bahsettiğim temele sahip insanlar gerekiyor. Çok ağır bir fizik yada matematik yok. Lise fiziğiyle bu problemin üstesinden geleceğiz. Bunun için sana güvenmiyorum yan çizeceksin gibime geliyor. Bu nedenle @teflon @Melon şapka yada matematik ve fizik bilgisine güvenebileceğimiz bize yönlendiren sorularıyla destek olabilecek kalitede belki @haci gibi daha farklı forum üyelerinden konuya destek gelirse daha iyi olabilir.

 

Zaten sorun senin lise düzeyinde bile bir matematik veya fizik bilgisine sahip olmamanda.

Jartiyer hikayelerin yerine senden sistemi bekliyoruz. Bilim forumunu bu şekilde meşgul edemezsin.

Sistemi yaptım diyorsan paylaş. Yorumu olan yapar. Fotoğrafını da istemiyoruz, biz forum olarak böyle bir kötülüğü haketmedik.

Link to post
Sitelerde Paylaş
2 saat önce, Bir Buçuk yazdı:

Sistemi yaptım diyorsan paylaş. Yorumu olan yapar. Fotoğrafını da istemiyoruz, biz forum olarak böyle bir kötülüğü haketmedik.

 

Endişe etmeyin bunu paylaşacağım ancak endişeliyim. Muhtemelen size öğretilen ve bilim sandığınız 20. yüzyıla ait bilgilerinize güvenerek "biri devridaimin olabileceğini iddia ediyorsa kesinlikle yanılıyordur" gibi bir kafaya sahip olmanızdan kaynaklı her türlü ön yargılarınızdan sıyrılmanızı talep ediyorum. Çünkü böyle düşünüyorsanız muhtemelen paylaşımımdan sonra çok fazla konuyla ilgili olmadan bir kaç kişinin paylaşacağı olumsuz geri dönüşlere güvenerek beni forumdan atmayı aklınızdan geçirebilirsiniz. Bunu hemen yapmayın. Öncelikle inceleyin ön yargısız davrandığınızdan emin olun ve ondan sonra çalışmayacağına kanaat getirirseniz forumdan atabilirsiniz fakat ani bir zanna kapılıp üzerine çok düşünmeden refleks olarak beni forumdan atmanızı istemem.

 

2 saat önce, Bir Buçuk yazdı:

Zaten sorun senin lise düzeyinde bile bir matematik veya fizik bilgisine sahip olmamanda.

 

Böyle düşünmenize sebep olan şey bir önceki paragrafta bahsettiklerim olabilir mi? Öyle ya koskoca bilim dünyası bir konuda yanılıyor olamaz değil mi? "Bir John Ahmet çıkıp tüm bunların aksine bir iddia ortaya koyuyorsa delinin tekidir" gibi düşüncelerinizden bilim yanlısı bir insan olduğunuzu iddia ettiğiniz kadar sıyrılamıyorsunuz değil mi? 

 

2 saat önce, Bir Buçuk yazdı:

Jartiyer hikayelerin yerine senden sistemi bekliyoruz. Bilim forumunu bu şekilde meşgul edemezsin.

 

Ben bunu konuya biraz mizah katarak size iddia ettiğim konunun ne kadar sivri bir iddia olduğunun farkında olduğumu ifade etmek için paylaştım. Yeterli sabrı, ciddiyeti ve dürüstlüğü gösterdiğinizde aklınıza sığdıramayacağınız bu konun ne kadar gerçek olduğunu anlamanız çok zor olacağı için en azından benim sabrıma yardımcı olacak söylemlerimi mizahtan başka bir şey olarak görmeyin.

 

 

________________________________________________________________________________________________________________________________________________________________

 

Sistemi anlatmaya başlayacağım ama öncelikle bu kısımdan sonrasına gerekli fizik ve matematik bilgilerine sahip olmayanların önyargı içeren ifadeleriyle başlığı kirletmemesi için sizden tam destek istiyorum. Ayrıca sistemin tümünü bir yazıda anlatmamı beklemeyin. Bu nedenle burada yalnızca konuya bir giriş yapacağım.

 

 

Tüm sistem, hava balonları ve pistonları ile su dolu büyükçe bir kabinin içerisindeki suyun kaldırma kuvveti ve pnömatik dinamiklerle çalışmaktadır. Öncelikle parçaları tanıtmam gerekir ve bunların dinamiğini incelememiz icap etmektedir fakat bunun üzerinde çok detaylı durmayacağım. Sadece konunun en temel noktasına değinip bunu anladığınızdan emin olmam gerekiyor.

 

Konuya çok basit bir matematikle çözüm bulabilmemiz için parçaları olduğundan daha basit geometrik objeler olarak ele alalım. Örneğin bir hava balonu ve pistonu olduğundan bahsetmiştim. Bir hava balonu düşünün. İçerisinde belirli bir miktar basınçta hava olan örneğin 10m derinliğe kadar deforme olmadan sağlam kalabilecek 10cm lik sert plastikten yapılmış tamamen kapalı bir küp düşünün. Bu pistonumuz olsun. Bunu suyu sızdırmayacak şekilde çevreleyen bir yüzü açık ikinci bir küpümüz daha olsun. Ayrıca bu yapı birbirlerinden ayrılmayacak şekilde birbirlerine sabitlenmiş olarak değil fakat kopup ayrılmayacak şekilde stoplanmış olsun. Bunlardan iki tane hazırlamış olalım ve bunlar havada iken pistonların içerisinde eşit miktarda hava kalacak biçimde açık vaziyetteyken birinin pistonu yukarıya bakacak şekilde diğerinin pistonu aşağıya bakacak şekilde tamamını içerisinde 1 m yükseklikte 1m3 su olan bir havuzun içerisine yerden 50 cm yükseklikte tutalım. 

 

Pistonların havadayken eşit olan hacimleri suyun içerisine daldırdığımızda nasıl değişiklik gösterir? Pistonu yukarıya bakana A, aşağıya bakana B dersek havadayken eşit basınç ve eşit hacimdeki piston yuvalarının hacimleri ile ilgili bağıntı suyun altında iken nasıl olur?

 

Dahası A ve B ye etki eden kaldırma kuvvetleri arasındaki bağıntıyı da eşitsizlik olarak ifade etmeniz mümkün müdür?

 

Bu arada pistonu yukarıya bakan şamandıranın maksimum hacimde olduğunu düşünün ve pistonun daha fazla bir hacim oluşmayacak şekilde stoplandığını düşünün. Dolayısıyla A'nın hacminde bir değişiklik meydana gelmeyecektir.

 

Öncelikle bu konuda ne düşündüğünüzden emin olmam gerekir ki daha sonra anlatacaklarımın tamamı bu prensibe göre şekillenecek. Konu ile ilgili olarak gerçek projede çok fazla detay vardır. Sizleri bu detaylarla meşgul etmeden konunun özünü anlatabilmem için bu soruya cevap vermenizi bekliyorum. 

 

Sistemin tamamını açıkladığımda beni anlayacağınızı umuyorum. Şimdilik buraya kadar olan kısmı iyi anladığınızdan emin olayım.

 

 

tarihinde John_Ahmet tarafından düzenlendi
Link to post
Sitelerde Paylaş

Tam netleştiremedim ama bazı durumları yazayım.

 

Aşa ve yukarı bakan pistonların yuvaları eşit seviyede ise, ve piston kafaları zorla yuvalarından çıkarılıp o halde kalmak için sabitlenmişse;

 

A, yani yukarı bakan piston ile B yani aşağıya bakan pistonun açık hallerine uygulanan kaldırma kuvveti ve basıncı inceleyelim. 

 

A ve B ye uygulanan kaldırma kuvvetleri eşittir. Tek bir pistonu ele alırsak, yanal yüzlere uygulanan basınçlar eşit olduğu için birbirini götürürken. Alt ve üst yüzeye uygulanan basınçlar derinlik farkı sebebiyle farklıdır. Üst yüzeye uygulanan basınç h ve alt yüzeye uygulanan basınç h' ise kaldırma kuvveti mantıken h'-h şeklinde gelir. 

 

Kaldırma kuvveti üstel bir foknkyona bağlı olarak değişmediği için, derinlik farkı h ve h' arasındaki farkı değiştirmeyecektir. Buda A ve B pistonlarına uygulanan kaldırma kuvvetinin eşit olduğunu gösterir. 

( 2 sininde tamamen suyun içinde olduğu var sayılmıştır) 

 

 

 

 

basınç için, daha derinde olan B pistonun açık kafası daha yüksek basınç altındadır. Ancak bükülmeye karşı dirençli olan ve hacmi değişmeyen plastik yüzünden, iç hava basıncı sabit kalmaktadır. 

Link to post
Sitelerde Paylaş
24 dakika önce, Sütlü Kase yazdı:

A ve B ye uygulanan kaldırma kuvvetleri eşittir.

 

A ve B pistonlarının tamamen kapalı birer küp olduklarını ve özellikle aşağıya bakan pistonun yaklaşık 10N'luk (ağırlık çok düşük şimdi görmezden gelebilirsin) bir kuvvetle pistonu saran ikinci küpün iç hacminde daha doğrusu değişken piston hacmi üzerinde değişiklik oluşturacağını dolayısıyla sistemin basınç altındaki toplam hacminin yukarıya bakandan daha farklı olacağını hesaba kattın mı? Bu arada A ve B ye sistem olarak etki eden kuvvetleri eş yükseklikte değerlendirerek cevap verebilir misin?

 

FKaldırma = Vbatan . g  su için düşünürsek yanlış bir formül kullanmıyorum öyle değil mi? Dolayısıyla kaldırma kuvveti hacme göre değişir yanlış mı düşünüyorum?

tarihinde John_Ahmet tarafından düzenlendi
Link to post
Sitelerde Paylaş
3 dakika önce, John_Ahmet yazdı:

 

A ve B pistonlarının tamamen kapalı birer küp olduklarını ve özellikle aşağıya bakan pistonun yaklaşık 10N'luk bir kuvvetle pistonu saran ikinci küpün iç hacminde daha doğrusu değişken piston hacmi üzerinde değişiklik oluşturacağını dolayısıyla sistemin basınç altındaki toplam hacminin yukarıya bakandan daha farklı olacağını hesaba kattın mı? Bu arada A ve B ye sistem olarak etki eden kuvvetleri eş yükseklikte değerlendirerek cevap verebilir misin?

 

Hiç bir şey anlamıyorum neyin neyi sardığı belli değil. Sıradan bir piston kafası ve kafanın oturduğu bir yuva hayal ederek cevap verdim. 

 

10N luk kuvvetle pistonu saran bir küp nasıl oluyor ? Lastik gibi bir şeyden mi bahsediyorsun, 10N nerden geldi ? Su basıncı mı ? 

 

Değişken piston hacmi nedir ? Hani sert plastikti ? 

 

Temel amacın ne ? Onu anlat şekli ben çıkarırım. 

Link to post
Sitelerde Paylaş

Tamam paintte şimdilik bunu çizdim. piston dediklerim daha doğrusu pistonun hareketli olanı 10x10x10 plastik kapalı içi hava dolu bir küp.

 

Diğeri ise küp yada prizma şeklindeki bir yüzeyinden pistonu içine alan dış kap. Dolayısıyla bu aradan su sızdırmayacak şekilde içerisinde piston hareket ediyor. sürtünme kuvveti olmadan düşün. Bu haliyle havada iken eşit V hacmine sahip eşit miktarda hava ile dolu. Bunları suyun içerisinde haraketli pistonlara dokunmadan dış kabından tutarak eş yükseklikte suda bekletiyoruz. Şimdi kaldırma kuvvetlerini hesaplamadan önce B ye ait kapalı küpün B'nin V hacmine olan etkisini de hesaba katarak kaldırma kuvvetleri olan Fve Fhakkında ne söyleyebilirsin? A ve B yi oluşturan iki parçaya etkiyen toplam kaldırma kuvvetini düşün.sekil-1.png

tarihinde John_Ahmet tarafından düzenlendi
Link to post
Sitelerde Paylaş

@Sütlü Kase şekli çizdim ve yeterince açıkladığımı düşünüyorum. Şimdi senden veya genel olarak sizlerden bir yorum bekliyorum. A ve B cismine etki eden Fa ve Fb kaldırma kuvvetleri hakkında neler söyleyebilirsin? Kaldırma kuvvetleri eşit midir?

tarihinde John_Ahmet tarafından düzenlendi
Link to post
Sitelerde Paylaş
54 dakika önce, John_Ahmet yazdı:

@Sütlü Kase şekli çizdim ve yeterince açıkladığımı düşünüyorum. Şimdi senden veya genel olarak sizlerden bir yorum bekliyorum. A ve B cismine etki eden Fa ve Fb kaldırma kuvvetleri hakkında neler söyleyebilirsin? Kaldırma kuvvetleri eşit midir?

 

Doğru hayal etmişim ve iletimde açıkladım. Pistonlar o şekilde kalmaya zorlanıyorsa, kaldırma kuvvetleri eşit etki eder. 

 

Serbest bırakırsan aşa bakan piston kafası kapanacaktır. ( kaldırma kuvettinin büyük olduğu var sayıldı) 

 

edit, tek farkım V hacimlerinin eşit seviyede olduğunu var saymıştım. Fark yapmaz ama 

tarihinde Sütlü Kase tarafından düzenlendi
Link to post
Sitelerde Paylaş
21 dakika önce, Sütlü Kase yazdı:

Doğru hayal etmişim ve iletimde açıkladım. Pistonlar o şekilde kalmaya zorlanıyorsa, kaldırma kuvvetleri eşit etki eder

 

Serbest bırakırsan aşa bakan piston kafası kapanacaktır. ( kaldırma kuvettinin büyük olduğu var sayıldı) 

 

Evet şu durumda FA nın daha büyük olduğunu kabul ediyoruz doğru mu anlamışım? Parantez içiresindeki cümlende "So what?" mı demeye getiyorsun yoksa son cümlende "kaldırma kuvvetinin büyük olduğu var sayıldı" derken "Fa'nın kaldırma kuvvetinin büyük olduğunu yükseklik parametresini netleştiremediğimden büyük olarak var sayıldı" diyorsun? O halde şuna kesin cevap verebilir misin? Makul derinlikler söz konusu olduğunda ve geometrik merkezleri eş yükseklikte ise A cisminin toplam hacmi daima B cisminin hacmimden fazla olur diyebilir misin? 

 

Elbette pistonları serbest bırakıyoruz yoksa neden sorayım veya ne işimize yarasın? Buna cevap verirsen bir sonraki aşamaya geçeceğiz.

 

V hacmin havadayken basıncını 1 atmosfer alabilirsin sakıncası yok. Zaten bir eşitsizlik problemi çözüyoruz.

tarihinde John_Ahmet tarafından düzenlendi
Link to post
Sitelerde Paylaş
16 dakika önce, John_Ahmet yazdı:

 

Evet şu durumda FA nın daha büyük olduğunu kabul ediyoruz doğru mu anlamışım? Parantez içiresindeki cümlende "So what?" mı demeye getiyorsun yoksa son cümlende "kaldırma kuvvetinin büyük olduğu var sayıldı" derken "Fa'nın kaldırma kuvvetinin büyük olduğunu yükseklik parametresini netleştiremediğimden büyük olarak var sayıldı" diyorsun? O halde şuna kesin cevap verebilir misin? Makul derinlikler söz konusu olduğunda ve geometrik merkezleri eş yükseklikte ise A cisminin toplam hacmi daima B cisminin hacmimden fazla olur diyebilir misin? 

 

Elbette pistonları serbest bırakıyoruz yoksa neden sorayım veya ne işimize yarasın? Buna cevap verirsen bir sonraki aşamaya geçeceğiz.

 

V hacmin havadayken basıncını 1 atmosfer alabilirsin sakıncası yok. Zaten bir eşitsizlik problemi çözüyoruz.

 

Pistonlar suyun içinde askıdalar mı ? Yoğunlukları sudan büyük mü küçük mü ? 

 

Buna cevap vererek devam et.

 

Zannediyorum ki derdin B pistonunun yuvasına geri sokan bir kuvvet oluşurken A pistonunu dışarda tutan bir kuvvet olmasını söylemek. 

 

 

 

 

Link to post
Sitelerde Paylaş
42 dakika önce, Sütlü Kase yazdı:

Zannediyorum ki derdin B pistonunun yuvasına geri sokan bir kuvvet oluşurken A pistonunu dışarda tutan bir kuvvet olmasını söylemek.

 

Buraya kadar sanırım beni anladın. Şimdi bu söyleyeceğim sistemin nihayî bir sonucu olduğunu düşünme ve şuna cevap verebilir misin bir bakalım.

 

Şimdi elimizde bu A yada B cisminden toplam 12 tanesini elimizdeki çember şeklindeki bir kayışa hepsi aynı yöne bakacak şekilde dış kabından sabitleyip bunu yukarıdaki ve aşağıdaki yaklaşık 20cm çapındaki iki daireli döner düzeneğimize birbirini bağlayan bir kayış olarak takalım ve böyle bir sistemi uygun büyüklükte bir cam kabin içerisine montajını sağlayalım ve özellikle tüm V hacimleri ve basınçları eşit olacak şekilde (T de sabit) ayarladıktan sonra cam kabinin içerisini tüm sistem su altında kalacak şekilde su ile dolduralım.

 

Sadece bir fikir olarak soruyorum. Bizim sistemin nihayî son hali bu değil elbete. Tüm pistonları birbirine bağlayan hava hortumları ve valflerin de olduğu daha farklı pnömatik sistemler söz  konusu ve tamamının logic kontrol sistemi var. Bu haliyle bu montajı yaptıktan ve suyu ilave ettikten sonra sistem kendiliğinden dönecek şekilde işler mi?

 

Şekili çok iyi çizemedim kabaca hepsinin eşit mesafede olduğunu düşün. Bu kesinlikle projenin son hali değildir bunu hiç denemedik ve sadece fikrinizi soruyorum. Çalışır mı? İşler mi? Neden Çalışmaz/Çalışır?

 

Sisteme bir yük bağlamak yada bağlamamak opsiyonel olsun. Bu durumu değiştirir mi? Neden?

 

sekil-2.png

 

 

 

 

tarihinde John_Ahmet tarafından düzenlendi
Link to post
Sitelerde Paylaş
17 dakika önce, John_Ahmet yazdı:

 

Buraya kadar beni anladığına kanaat getirdim. Şimdi bu söyleyeceğimi sistemin nihayî bir sonucu olduğunu düşünme ve şuna cevap verebilir misin bir bakalım.

 

Şimdi elimizde bu A yada B cisminden toplam 12 tanesini elimizdeki çember şeklindeki bir kayışa hepsi aynı yöne bakacak şekilde dış kabından sabitleyip bunu yukarıdaki ve aşağıdaki yaklaşık 20cm çapındaki iki daireli döner düzeneğimize birbirini bağlayan bir kayış olarak takalım ve böyle bir sistemi uygun büyüklükte bir cam kabin içerisine montajını sağlayalım ve özellikle tüm V hacimleri ve basınçları eşit olacak şekilde (T de sabit) ayarladıktan sonra cam kabinin içerisini tüm sistem su altında kalacak şekilde su ile dolduralım.

 

Sadece bir fikir olarak soruyorum. Bizim sistemin nihayî son hali bu değil elbete. Tüm pistonları birbirine bağlayan hava hortumları ve valflerin de olduğu daha farklı pnömatik sistemler söz  konusu ve tamamının logic kontrol sistemi var. Bu haliyle bu montajı yaptıktan ve suyu ilave ettikten sonra sistem kendiliğinden dönecek şekilde işler mi?

 

Şekili çok iyi çizemedim kabaca hepsinin eşit mesafede olduğunu düşün. Bu kesinlikle projenin son hali değildir bunu hiç denemedik ve sadece fikrinizi soruyorum. Çalışır mı? İşler mi? Neden Çalışmaz/Çalışır?

 

Sisteme bir yük bağlamak yada bağlamamak opsiyonel olsun bu durumu değiştirir mi? Neden?

 

sekil-2.png

 

 

 

 

Hayır dönmez, 

 

Öncelikle sistemin dönmesini nasıl bekleriz onu inceleyelim. Sonrada hata noktalarına bakalım. ( bunu doğru anlamışmıyım bak diye yazıyorum)

 

Öncelikle pistonların kapalı halleri suda batarken açık halleri suda batmayacak şekilde ayarlanmıştır. 

 

Yukarıdaki sisteme bakarken sol taraftaki açık pistonlar yükselmek isteyecek, beraberinde dönebilen bir çarka sarımlı kayışı yukarı çekecektir.

Sistem belirli bir miktar dönecektir. 

 

Pistonlar kayışa bağlı olduğu için dairesel bir hareket çizip, ilk hız avantajını da kullanarak baş aşağı konuma gelecektir. Tam bu anda hala yükselmek isteyen pistonların açık kısımları, serbest şekilde hareket ettikleri yuvaya geri girecek. Buda kaldırma kuvvetini düşürerek pistonun batmasını ve kayışın aşa yönlü hareketine sebep olacak. 

 

Piston tekrar sol tarafa geldiğinde, kaldırma kuvveti etkisiyle yukarı doğru açılacak ve buda yükselmesini sağlayacak. 

 

Bu sisten sürekli olarak tekrarlayacak 

 

 

Doğru muyum ???? 

Link to post
Sitelerde Paylaş
19 dakika önce, Sütlü Kase yazdı:

Doğru muyum ????

 

Evet gayet açık.

 

Vsol toplam > Vsağ toplam

 

ise daima döner hatta kuvvet oluşturacak şekilde döner öyle değil mi? Peki gerçekten öyle midir? Sol taraftaki hacim toplamı sağ taraftakilerden büyük ise döner diyebiliriz o halde değil mi?

 

Sistemin temel elemanı olan şamandırayı anladık öyle değil mi? Bunlar küp değil de silindir de olabilir yada sızdırmazlık sağlanamazsa eş kapasite hacimli körük de kullanılabilir vs.

 

Pistonların hareketli iç hacimlerine uygun basınçta hava ile doldurduğumuzu varsayıyoruz 1 atmosfer değil de 2 olur vs. Uygun olanı düşün.

 

Şimdi bunun hakkında ne söyleyebilirsin? Net bir şey söyleyebilir misin? Termodinamik var o halde çalışmaz dışında gerçek bir matematik hesapla çalışmayacaklarını gösterebilir misin? Senin fikrin ne yönde çalışır mı? Çalışmaz mı? Fakat neden olduğunu da açıklamak koşulu ile neler söyleyebilirsin? 

 

Sistemde içine su girecek ucu açık bir piston yok farkında mısın sadece basınçla hacmi değişen bu yapıdaki pistonların her biri dış kabına kopup ayrılmasın diye dışarıdan birbirlerine iple bağlı şeklinde düşünebilirsin.

 

Çalışır mı? Neden?

tarihinde John_Ahmet tarafından düzenlendi
Link to post
Sitelerde Paylaş
9 dakika önce, John_Ahmet yazdı:

 

Evet gayet açık.

 

Vsol toplam > Vsağ toplam

 

ise daima döner hatta kuvvet oluşturacak şekilde döner öyle değil mi? Peki gerçekten öyle midir? Sol taraftaki hacim toplamı sağ taraftakilerden büyük ise döner diyebiliriz o halde değil mi?

 

Sistemin temel elemanı olan şamandırayı anladık öyle değil mi? Bunlar küp değil de silindir de olabilir yada sızdırmazlık sağlanamazsa eş kapasite hacimli körük de kullanılabilir vs.

 

Pistonların hareketli iç hacimlerine uygun basınçta hava ile doldurduğumuzu varsayıyoruz 1 atmosfer değil de 2 olur vs. Uygun olanı düşün.

 

Şimdi bunun hakkında ne söyleyebilirsin? Net bir şey söyleyebilir misin? Termodinamik var ohalde çalışmaz dışında gerçek bir matematik hesapla çalışmayacaklarını gösterebilir misin? Senin fikrin ne yönde çalışır mı? Çalışmaz mı? Fakat neden olduğunu da açıklamak koşulu ile neler söyleyebilirsin? 

 

Sistemde içine su girecek ucu açık bir piston yok farkında mısın sadece basınçla hacmi değişen bu yapıdaki pistonların her biri dış kabına kopup ayrılmasın diye dışarıdan birbirlerine iple bağlı şeklinde düşünebilirsin.

 

Çalışır mı? Neden?

Güzel, anlaşmışız. 

 

Şimdi çalışmama sebebine gelelim. 

 

1-) 

 

Kaldırma kuvveti cismin her zaman kütle merkezine etki eder. Sistem bir tam tur döndükten sonra sol tarafta kapalı kalan piston açılmaz. Ortada pistonu açacak bir kuvvet yok. 

 

Kuvvet diyagramı çizersem. Kapalı pistonun her yerine iç yönlü ( pistonun ağırlık merkezine doğru) oklar koymam gerekir. Yanal oklar sadeleşirken alt ve üstteki oklar derinlik farkı sebebiyle sadeleşmez... Hatta alt taraftaki okların daha büyük olduğu ( kuvvetin daha büyük olduğu) ortadadır buda zaten kaldırma kuvvetinin tanımıdır. 

 

Bu tanıma göre kapalı pistonu açacak bir kuvvet yoktur. Sistem takılır. 

 

 

 

 

 

Yazmaya devam edecektim ki buna olan cevabını görmek istiyorum önce 

tarihinde Sütlü Kase tarafından düzenlendi
Link to post
Sitelerde Paylaş
17 dakika önce, Sütlü Kase yazdı:

Bu tanıma göre kapalı pistonu açacak bir kuvvet yoktur. Sistem takılır. 

 

 

 

 

 

Yazmaya devam edecektim ki buna olan cevabını görmek istiyorum önce

 

Her pistonun hem hacim farkını oluşturan kuvveti sağlayan kapalı pistonun ve kabı ile arasındaki hava sabittir değişmiyor ve şu haliyle hiç bir yere hava kaçırmıyor. Piston en aşağıda neden kapanıp bir daha açılamasın anlamadım. Şimdi yanlış anlamış gibi yapıp yan mı çizeceksin? Açık piston ve onu kapatan içi hava dolu piston tam olarak kapalı bir sistemdir. Hacmi sadece basınç değişimi olduğunda değişir. En alttaki basıncın onun hacmini sıfırlayacağını neden düşündün hava bir yerlere kaçmıyor ki? Dolayısıyla su derinliği değişince basınç farkı oluşuyor fakat sol tarafa geçtiklerinde hacmi eş yükseliğe göre daha büyük olacak şekilde kapalı olan pistonun kaldırma kuvveti nedeniyle toplam hacim değişecektir. Objeler su içerisinde maksimum hızlarında hareket etmiyor bu da ayrı bir detaydır ve fark varsa yük bağlanabilir bağlanırsa hız düşer. Burası çok önemlidir.

 

Hepsi fazlası ile suyun altında gerçekleşiyor açık aramayın.

tarihinde John_Ahmet tarafından düzenlendi
Link to post
Sitelerde Paylaş
  • Konuyu Görüntüleyenler   0 kullanıcı

    Sayfayı görüntüleyen kayıtlı kullanıcı bulunmuyor.


Kitap

Yazar Ateistforum'un kurucularındandır. Kitabı edinme seçenekleri için: Kitabı edinme seçenekleri

Ateizmi Anlamak
Aydın Türk
Propaganda Yayınları; / Araştırma
ISBN: 978-0-9879366-7-7


×
×
  • Yeni Oluştur...